1961 IMO Problems/Problem 4

Revision as of 11:33, 12 October 2007 by 1=2 (talk | contribs)

Problem

In the interior of triangle $ABC$ a point P is given. Let $Q_1,Q_2,Q_3$ be the intersections of $PP_1, PP_2,PP_3$ with the opposing edges of triangle $ABC$. Prove that among the ratios $\frac{PP_1}{PQ_1},\frac{PP_2}{PQ_2},\frac{PP_3}{PQ_3}$ there exists one not larger than 2 and one not smaller than 2.

Solution

This problem needs a solution. If you have a solution for it, please help us out by adding it.


See Also

1961 IMO Problems